Rozkład Poissona

Otrzymałeś(aś) rozwiązanie do zamieszczonego zadania? - podziękuj autorowi rozwiązania! Kliknij
Klaudia23
Dopiero zaczynam
Dopiero zaczynam
Posty: 22
Rejestracja: 20 sty 2020, 16:21
Podziękowania: 5 razy
Płeć:

Rozkład Poissona

Post autor: Klaudia23 »

Zmienna losowa ma rozkład Poissona , gdzie γ3 = 0.43 (gamma()
Proszę obliczyć:
-Dominantę
-Odchylenie standardowe
- γ4(gamma)
-Kwartyl1
-Medianę
-P(X=2,9)
-P(X<3)
-P(X≥5)
-P(2⩽X ⩽4)
Awatar użytkownika
panb
Expert
Expert
Posty: 5122
Rejestracja: 26 kwie 2010, 22:54
Lokalizacja: Nowiny Wielkie
Podziękowania: 19 razy
Otrzymane podziękowania: 2053 razy
Płeć:

Re: Rozkład Poissona

Post autor: panb »

Co to są te gammy? \(\gamma_3,\,\,\, \gamma_4\)?
Klaudia23
Dopiero zaczynam
Dopiero zaczynam
Posty: 22
Rejestracja: 20 sty 2020, 16:21
Podziękowania: 5 razy
Płeć:

Re: Rozkład Poissona

Post autor: Klaudia23 »

panb pisze: 22 sty 2020, 00:31 Co to są te gammy? \(\gamma_3,\,\,\, \gamma_4\)?
Trzeci i czwarty moment centralny standaryzowany
Awatar użytkownika
panb
Expert
Expert
Posty: 5122
Rejestracja: 26 kwie 2010, 22:54
Lokalizacja: Nowiny Wielkie
Podziękowania: 19 razy
Otrzymane podziękowania: 2053 razy
Płeć:

Re: Rozkład Poissona

Post autor: panb »

OK, u mnie one nazywały się \(\mu_k\).
Awatar użytkownika
panb
Expert
Expert
Posty: 5122
Rejestracja: 26 kwie 2010, 22:54
Lokalizacja: Nowiny Wielkie
Podziękowania: 19 razy
Otrzymane podziękowania: 2053 razy
Płeć:

Re: Rozkład Poissona

Post autor: panb »

Policzę parametr \(\lambda\) rozkładu, dalej to już powinno być łatwo...
  1. Fakt: \( \sum_{n=0}^{ \infty } \frac{\lambda^n}{n!}=e^\lambda \)
  2. \( \sum_{n=0}^{ \infty }n \cdot \frac{\lambda^n}{n!}= \sum_{n=1}^{ \infty } \frac{\lambda^n}{n!}=\lambda\sum_{n=1}^{ \infty } \frac{\lambda^{n-1}}{(n-1)!} = \begin{vmatrix} k=n-1\\ n=1 \So k=0\end{vmatrix} =\lambda\sum_{k=0}^{ \infty } \frac{\lambda^k}{k!} =\lambda e^\lambda\)
  3. \(\sum_{n=0}^{ \infty }n^2 \frac{\lambda^n}{n!}=\lambda\sum_{n=1}^{ \infty } n \frac{\lambda^{n-1}}{(n-1)!}= \begin{vmatrix} k=n-1\\n=k+1\\n=1 \So k=0\end{vmatrix} =\lambda\sum_{k=0}^{ \infty }(k+1) \frac{\lambda^k}{k!} =\lambda \left(\sum_{k=0}^{ \infty } k \frac{\lambda^k}{k!}+\sum_{k=0}^{ \infty } \frac{\lambda^k}{k!}\right)=\\=\lambda \cdot \lambda e^\lambda+\lambda e^\lambda =\lambda e^\lambda(\lambda+1) \)
  4. \(\sum_{n=0}^{ \infty }n^3 \frac{\lambda^n}{n!}=\lambda\sum_{n=1}^{ \infty }n^2 \frac{\lambda^{n-1}}{(n-1)!} =\lambda\sum_{k=0}^{ \infty }(k+1)^2 \frac{\lambda^k}{k!}=\lambda \left(\sum_{k=0}^{ \infty }k^2 \frac{\lambda^k}{k!}+2\sum_{k=0}^{ \infty }k \frac{\lambda^k}{k!} +\sum_{k=0}^{ \infty } \frac{\lambda^k}{k!} \right) =\\=\lambda \left(\lambda(\lambda+1)e^\lambda+2\lambda e^\lambda +e^\lambda\right) =e^\lambda \left(\lambda^3+3\lambda^2+\lambda \right) \)
Teraz \(\gamma_3=\sum_{n=0}^{ \infty }(n-\lambda)^3 \frac{\lambda^n}{n!}e^{-\lambda}=e^{-\lambda} \left( \sum_{n=0}^ { \infty }(n^3-3n^2\lambda +3n\lambda^2+\lambda^3\right) \frac{\lambda^n}{n!} ) \)

Korzystając z obliczeń i - iv , otrzymamy \(\gamma_3=\lambda=0,43\)

Teraz można już spokojnie policzyć większość potrzebnych rzeczy (a na pewno wszystkie P(...) )
ODPOWIEDZ